Show charge conservation in a curved spacetime

Actually, I think I made a mistake. That formula is just the trace of the Christoffel symbol. But it does lead to a simpler proof of the formula you're trying to prove. Let me write it up and I'll post it.In summary, we can use the antisymmetry of ##F^{ab}## and the identities ##\nabla_a J^a = \frac{1}{\sqrt{|g|}} \partial_a \left( \sqrt{|g|} J^a \right)## and ##\nabla_a F^{ab} = \frac{1}{\sqrt{|g|}} \partial_a \left( \sqrt{|g|} F^{ab}
  • #1
etotheipi
Homework Statement
Given Maxwell's equations ##\nabla_a F^{ab} = -4\pi j^b## and ##\nabla_{[a}F_{bc]} = 0##
Relevant Equations
N/A
For the flat spacetime we could just use that partial derivatives commute as well as the antisymmetry of ##F^{ab}##, i.e. ##\partial_b \partial_a F^{ab} = -\partial_b \partial_a F^{ba} = -\partial_a \partial_b F^{ba} = -\partial_b \partial_a F^{ab} \implies \partial_b \partial_a F^{ab} = - 4\pi \partial_b j^b = 0 \implies \partial_b j^b = 0##.

The covariant derivative doesn't necessarily commute, I can only get ##\left[\nabla_b \nabla_a + \nabla_a \nabla_b\right] F^{ab} = 0## from the antisymmetry of ##F^{ab}##. But I tried to use the Riemann tensor to write down$$[\nabla_b \nabla_a - \nabla_a \nabla_b]F^{cd} = -{R_{bae}}^{c} F^{ed} - {R_{bae}}^d F^{ce}$$ $$\nabla_a \nabla_b F^{ab} = \nabla_b \nabla_a F^{ab} + {R_{bae}}^a F^{eb} + {R_{bae}}^b F^{ae}$$So we have$$0 = \left[\nabla_b \nabla_a + \nabla_a \nabla_b\right] F^{ab} = 2\nabla_b \nabla_a F^{ab} + {R_{bae}}^a F^{eb} + {R_{bae}}^b F^{ae} = 2\nabla_b \nabla_a F^{ab} + R_{be} F^{eb} - {R_{be}} F^{be}$$where in the last term I relabelled the dummy indices from ##a## to ##b##. Why aren't the last two terms vanishing, instead they sum to ##R_{be} F^{eb} - {R_{be}} F^{be} = 2R_{be} F^{eb}##? Thanks!
 
  • Like
Likes Hamiltonian, JD_PM and vanhees71
Physics news on Phys.org
  • #2
etotheipi said:
$$0 = \left[\nabla_b \nabla_a + \nabla_a \nabla_b\right] F^{ab} = 2\nabla_b \nabla_a F^{ab} + {R_{bae}}^a F^{eb} + {R_{bae}}^b F^{ae} = 2\nabla_b \nabla_a F^{ab} + R_{be} F^{eb} - {R_{be}} F^{be}$$where in the last term I relabelled the dummy indices from ##a## to ##b##. Why aren't the last two terms vanishing?
Does ##R_{a b}## have any symmetry properties?
 
Last edited:
  • Like
  • Love
Likes vanhees71, JD_PM and etotheipi
  • #3
Ah, that's it! The properties of the Riemann tensor are, ##{R_{abc}}^d = - {R_{bac}}^d##, and secondly ##{R_{[abc]}}^d = 0##. Thirdly, if the covariant derivative is chosen to be the one that is metric compatible (i.e. ##\nabla_a g_{bc} = 0##), then ##R_{abcd} = -R_{abdc}##, and finally we have the Bianchi identity ##\nabla_{[a} {R_{bc]d}}^e##.

Let's start then with the second property,$$\frac{1}{6} \left[ {R_{abc}}^d - {R_{acb}}^d + {R_{bca}}^d - {R_{bac}}^d + {R_{cab}}^d - {R_{cba}}^d \right] = 0$$Now we use the antisymmetry in the first two arguments (property 1), multiply out the irrelevant numerical factors,$$\begin{align*}{R_{abc}}^d + {R_{bca}}^d + {R_{cab}}^d = 0 \implies {R_{abc}}^d - {R_{cba}}^d + {R_{cab}}^d

\end{align*}$$After re-writing the second term, then letting ##d=b## and summing over ##b##,$$R_{ac} - R_{ca} + {R_{cab}}^b = 0$$However, since$${R_{cab}}^b = g^{be}R_{cabe} = -g^{be}R_{caeb} = - {R_{cae}}^{e} = - {R_{cab}}^{b}$$we must have ##{R_{cab}}^b=0##, and thus$$R_{ac} = R_{ca}$$Hence, going back to the question,$$\begin{align*}0 = 2\nabla_b \nabla_a F^{ab} + R_{be} F^{eb} - {R_{be}} F^{be} &= 2\nabla_b \nabla_a F^{ab} + R_{be} F^{eb} - {R_{eb}} F^{be} \\

&= 2\nabla_b \nabla_a F^{ab} + R_{be} F^{eb} - {R_{be}} F^{eb} = 2\nabla_b \nabla_a F^{ab}

\end{align*}$$And thus ##\nabla_b j^b = 0##, as required. Thanks again, I hope the above is alright!
 
  • #4
That looks good.

An alternate approach to showing that ##\nabla_b J^b = 0## follows from ##\nabla_a F^{ab} = -4 \pi J^b## is to use a couple of well-known identities regarding divergences. These identities are derived in the standard texts.

For any vector ##J^a##, $$\nabla_a J^a = \frac{1}{\sqrt{|g|}}\partial_a \left(\sqrt{|g|} \, J^a \right) \qquad \qquad [1]$$

For any antisymmetric tensor ##F^{ab}##, $$\nabla_a F^{ab} = \frac{1}{\sqrt{|g|}}\partial_a \left(\sqrt{|g|} \, F^{ab} \right) \qquad \quad[2]$$

You don't need to invoke the curvature tensor if you use these.
 
  • Informative
  • Like
Likes Pouramat, etotheipi and JD_PM
  • #5
Quick question: was this exercise aimed at showing that ##\nabla_{\mu} j^{\mu} = 0## by means of Maxwell's equations (i.e. ##\nabla_{\mu} F^{\mu \nu} = -4\pi j^{\nu}##?
 
  • Like
Likes etotheipi
  • #6
TSny said:
That looks good.

An alternate approach to showing that ##\nabla_b J^b = 0## follows from ##\nabla_a F^{ab} = -4 \pi J^b## is to use a couple of well-known identities regarding divergences. These identities are derived in the standard texts.

For any vector ##J^a##, $$\nabla_a J^a = \frac{1}{\sqrt{|g|}}\partial_a \left(\sqrt{|g|} \, J^a \right) \qquad \qquad [1]$$

For any antisymmetric tensor ##F^{ab}##, $$\nabla_a F^{ab} = \frac{1}{\sqrt{|g|}}\partial_a \left(\sqrt{|g|} \, F^{ab} \right) \qquad \quad[2]$$

You don't need to invoke the curvature tensor if you use these.

Would that be,$$\nabla_b J^b = \frac{1}{\sqrt{|g|}} \partial_b \left( -\frac{\sqrt{|g|}}{4\pi} \cdot \frac{1}{\sqrt{|g|}} \partial_a \left( \sqrt{|g|} F^{ab}\right) \right) = -\frac{1}{4\pi} \partial_b \partial_a F^{ab} = 0$$where I needed to use that ##\partial_a \sqrt{|g|} \equiv \sqrt{|g|} \partial_a##, because the determinant of the metric doesn't depend on the coordinates.

That's pretty cool!

JD_PM said:
Quick question: was this exercise aimed at showing that ##\nabla_{\mu} j^{\mu} = 0## by means of Maxwell's equations (i.e. ##\nabla_{\mu} F^{\mu \nu} = -4\pi j^{\nu}##?

Yeah, it's just that. The more advanced exercise might be to do it from Noether's theorem, but I don't know how do that yet!
 
  • Like
Likes JD_PM
  • #7
If you wish to prove TSny ##[1]## formula, you may need to show first that the Christoffel symbol takes the form

$$\Gamma_{\mu \nu}^{\mu} =\frac{1}{\sqrt{|g|}}\partial_{\nu} \left(\sqrt{|g|}\right)$$
 
  • Like
  • Informative
Likes TSny and etotheipi
  • #8
Okay, I'm a bit stuck. I did the easy part,$$\begin{align*}

\Gamma^{\mu}_{\mu \nu} &= \frac{1}{2} g^{\mu \rho}(\partial_{\mu} g_{\nu \rho} + \partial_{\nu} g_{\mu \rho} - \partial_{\rho} g_{\mu \nu}) \\

&= \left[ \frac{1}{2} g^{\mu \rho}\partial_{\mu} g_{\nu \rho} - \frac{1}{2} g^{\rho \mu} \partial_{\mu} g_{\rho \nu} \right] + \frac{1}{2} g^{\mu \rho} \partial_{\nu} g_{\mu \rho} = \frac{1}{2} g^{\mu \rho} \partial_{\nu} g_{\mu \rho}

\end{align*}$$but how do I get that into the form of the determinant? Maybe I need to go see if Tong mentioned it anywhere 🤭
 
  • Like
Likes JD_PM
  • #9
Oh, Tong did mention it on page 111 of his third set of notes, the identity$$\text{tr}(\text{log}(A)) = \text{log}(\text{det}(A))$$for any diagonalisable matrix ##A##
 
  • Like
Likes JD_PM
  • #10
etotheipi said:
Would that be,$$\nabla_b J^b = \frac{1}{\sqrt{|g|}} \partial_b \left( -\frac{\sqrt{|g|}}{4\pi} \cdot \frac{1}{\sqrt{|g|}} \partial_a \left( \sqrt{|g|} F^{ab}\right) \right) = -\frac{1}{4\pi} \partial_b \partial_a F^{ab} = 0$$where I needed to use that ##\partial_a \sqrt{|g|} \equiv \sqrt{|g|} \partial_a##, because the determinant of the metric doesn't depend on the coordinates.
That's almost it. But, in general, ##g## depends on the coordinates. For example, using spherical coordinates in Minkowski spacetime, ##g = -r^4 \sin^2 \theta##.

So, you cannot "factor out" ##\sqrt{|g|}## in the expression ##\partial_a \left( \sqrt{|g|} F^{ab}\right)##. But you should still be able to get the result.
 
  • Informative
Likes etotheipi
  • #11
etotheipi said:
Oh, Tong did mention it on page 111 of his third set of notes, the identity$$\text{tr}(\text{log}(A)) = \text{log}(\text{det}(A))$$for any diagonalisable matrix ##A##
Okay, in that case,$$\begin{align*}

\nabla_a J^a = \partial_a J^a + J^c \Gamma^{a}_{ac} &= \partial_a J^a + \frac{J^a}{\sqrt{|g|}} \partial_a \left(\sqrt{|g|} \right) \\

&= \frac{1}{\sqrt{|g|}} \left[ \sqrt{|g|}\partial_a J^a + J^a \partial_a \left(\sqrt{|g|} \right)\right] \\

&= \frac{1}{\sqrt{|g|}}\partial_a \left(\sqrt{|g|} J^a \right)

\end{align*}$$🥳
 
Last edited by a moderator:
  • Like
Likes TSny
  • #12
TSny said:
That's almost it. But, in general, ##g## depends on the coordinates. For example, using spherical coordinates in Minkowski spacetime, ##g = -r^4 \sin^2 \theta##.

So, you cannot "factor out" ##\sqrt{|g|}## in the expression ##\partial_a \left( \sqrt{|g|} F^{ab}\right)##. But you should still be able to get the result.

That makes sense! I tried$$\partial_b \partial_a \left( F^{ab} \sqrt{|g|} \right) = \partial_b \left( \sqrt{|g|} \right) \partial_a F^{ab} + \sqrt{|g|} \partial_b \partial_a F^{ab} + \partial_a \left( \sqrt{|g|} \right) \partial_b F^{ab} + F^{ab} \partial_b \partial_a \left( \sqrt{|g|} \right)$$The second term on the RHS is zero, whilst the first and third terms sum to $$\begin{align*}

\partial_b \left( \sqrt{|g|} \right) \partial_a F^{ab} + \partial_a \left( \sqrt{|g|} \right) \partial_b F^{ab} &=

\partial_b \left( \sqrt{|g|} \right) \partial_a F^{ab} + \partial_b \left( \sqrt{|g|} \right) \partial_a F^{ba} \\

&= \partial_b \left( \sqrt{|g|} \right) \partial_a F^{ab} - \partial_b \left( \sqrt{|g|} \right) \partial_a F^{ab} = 0
\end{align*}$$and thus$$\partial_b \partial_a \left( F^{ab} \sqrt{|g|} \right) = F^{ab} \partial_b \partial_a \left( \sqrt{|g|} \right)$$However, $$F^{ab} \partial_b \partial_a \left( \sqrt{|g|} \right) = - F^{ba} \partial_b \partial_a \left( \sqrt{|g|} \right) = - F^{ba} \partial_a \partial_b \left( \sqrt{|g|} \right) = - F^{ab} \partial_b \partial_a \left( \sqrt{|g|} \right)$$so ´##F^{ab} \partial_b \partial_a \left( \sqrt{|g|} \right) = 0## and thus ##\partial_b \partial_a \left( F^{ab} \sqrt{|g|} \right) = 0##.
 
  • Like
Likes JD_PM and vanhees71
  • #13
$$\partial_b \partial_a \left( F^{ab} \sqrt{|g|} \right) $$

Note that the quantity inside the parentheses is antisymmetric in ##a## and ##b##.
 
  • Like
  • Wow
Likes JD_PM and etotheipi
  • #14
TSny said:
$$\partial_b \partial_a \left( F^{ab} \sqrt{|g|} \right) $$

Note that the quantity inside the parentheses is antisymmetric in ##a## and ##b##.

Ah, whoops 🤭, well it was good practice anyway 😆. Yeah, I probably should have seen that. Oh well. Thanks for explaining the second approach to the problem, the more methods the merrier!
 
  • Like
Likes JD_PM and TSny
  • #15
etotheipi said:
Okay, I'm a bit stuck. I did the easy part,$$\begin{align*}

\Gamma^{\mu}_{\mu \nu} &= \frac{1}{2} g^{\mu \rho}(\partial_{\mu} g_{\nu \rho} + \partial_{\nu} g_{\mu \rho} - \partial_{\rho} g_{\mu \nu}) \\

&= \left[ \frac{1}{2} g^{\mu \rho}\partial_{\mu} g_{\nu \rho} - \frac{1}{2} g^{\rho \mu} \partial_{\mu} g_{\rho \nu} \right] + \frac{1}{2} g^{\mu \rho} \partial_{\nu} g_{\mu \rho} = \frac{1}{2} g^{\mu \rho} \partial_{\nu} g_{\mu \rho}

\end{align*}$$but how do I get that into the form of the determinant? Maybe I need to go see if Tong mentioned it anywhere 🤭

Good job, you are halfway!

etotheipi said:
Oh, Tong did mention it on page 111 of his third set of notes, the identity$$\text{tr}(\text{log}(A)) = \text{log}(\text{det}(A))$$for any diagonalisable matrix ##A##

Indeed, we need to use Jacobi's identity to prove the formula at #7. I suggest you massage it a bit, so that it is easier to apply to our specific problem.

Let us use the chain rule on the RHS

\begin{equation*}
\frac{1}{\sqrt{|g|}}\partial_{\nu} \left(\sqrt{|g|}\right) = \frac{1}{\sqrt{|g|}}\left( \frac{1}{2 \sqrt{|g|}}\partial_{\nu} g \right) = \frac{1}{2g} \partial_{\nu} g
\end{equation*}

Now we make the following observations: the metric ##g_{\mu \nu}## is an invertible square matrix ##\text{M}## (i.e. ##g^{\mu \rho}g_{\rho \nu}=\delta_{\nu}^{\mu}##), the derivative is a linear operator ##\text{D}## and the determinant of the metric is ##\sqrt{|g|}=\det(g_{\mu \nu}):=g##.

As you pointed out, at this stage we need Jacobi's identity. Let us though give it in a more convenient form

\begin{equation*}
\text{D} \det(\text{M}) = \det(\text{M}) tr(\text{M}^{-1} \text{D} \text{M})
\end{equation*}

Applying it to our particular problem we get

\begin{align*}
\partial_{\nu} g &= g tr(g^{\mu \rho} \partial_{\nu} g_{\rho \sigma}) \\
&= g g^{\mu \rho} \partial_{\nu} g_{\rho \mu}
\end{align*}

Thus we get

\begin{equation*}
\frac{1}{\sqrt{|g|}}\partial_{\nu} \left(\sqrt{|g|}\right) = \frac{1}{2g} g g^{\mu \rho} \partial_{\nu} g_{\rho \mu}=\frac{1}{2} g^{\mu \rho} \partial_{\nu} g_{\rho \mu}
\end{equation*}

Notice that what we have just obtained is precisely the same that you got at #8. Thus we only have to equate both equations i.e.

\begin{equation*}
\Gamma^{\mu}_{\mu \nu} = \frac{1}{2} g^{\mu \rho} \partial_{\nu} g_{\rho \mu} = \frac{1}{\sqrt{|g|}}\partial_{\nu} \left(\sqrt{|g|}\right)
\end{equation*}

QED.
 
  • Like
Likes etotheipi
  • #16
Still hungry? Here's a quick snack!

Show that the Klein Gordon operator (i.e. ##\Box := \nabla_{\mu}\nabla^{\mu}##), when applied to a scalar field ##\phi(x)##, takes the following form

\begin{equation*}
\Box \phi = \frac{1}{\sqrt{|g|}} \partial_{\mu} \left(\sqrt{|g|} g^{\mu \nu} \partial_{\nu} \phi \right)
\end{equation*}

The approach is very similar to the one you used to show TSny ##[1]## formula. You will also need to use the Christoffel symbol formula we have just proven
 
  • Like
Likes etotheipi and vanhees71
  • #17
Another hint (much less tedious). Use the action principle for the free KG field. In flat space the action reads
$$A=\int \mathrm{d}^4 x \frac{1}{2} (\partial_{\mu} \phi)(\partial^{\mu} \phi)-\frac{m}{2} \phi^2.$$
Just write it in a generally covariant form for arbitrary spacetime coordinates ##q^{\mu}## and then evaluate the Euler-Lagrange equation.
 
  • Like
  • Informative
Likes JD_PM and etotheipi
  • #18
JD_PM said:
Show that the Klein Gordon operator (i.e. ##\Box := \nabla_{\mu}\nabla^{\mu}##), when applied to a scalar field ##\phi(x)##, takes the following form

\begin{equation*}
\Box \phi = \frac{1}{\sqrt{|g|}} \partial_{\mu} \left(\sqrt{|g|} g^{\mu \nu} \partial_{\nu} \phi \right)
\end{equation*}

I tried, given the covariant derivative of a scalar field is just a partial derivative, and using the result we already worked for @TSny's equation [1], $$\begin{align*}

\nabla_{\mu} \nabla^{\mu} \phi = \nabla_{\mu} \partial^{\mu} \phi &= \frac{1}{\sqrt{|g|}} \partial_{\mu} \left(

\sqrt{|g|} \partial^{\mu} \phi \right) \\

&= \frac{1}{\sqrt{|g|}} \partial_{\mu} \left(\sqrt{|g|} g^{\mu \nu} \partial_{\nu} \phi \right)\end{align*}$$
Also, one notational question, do we define ##g \equiv \text{det}[(g_{\mu \nu})]##, and then I guess ##|g|## would just be the modulus, which we need to use when we have a Lorentzian metric?
vanhees71 said:
Another hint (much less tedious). Use the action principle for the free KG field. In flat space the action reads$$A=\int \mathrm{d}^4 x \frac{1}{2} (\partial_{\mu} \phi)(\partial^{\mu} \phi)-\frac{m}{2} \phi^2.$$
Just write it in a generally covariant form for arbitrary spacetime coordinates ##q^{\mu}## and then evaluate the Euler-Lagrange equation.
I haven't learned anything about field theory so I'm probably going to get a lot of this wrong, but here's what I tried. I don't know what it means to write it in general covariant form, so instead I just tried to see what I could do in the special case that ##\nabla \equiv \partial##...$$\mathcal{L} = \frac{1}{2} g^{\mu \nu} (\partial_{\mu} \phi) (\partial_{\nu} \phi) - \frac{m}{2} \phi^2$$Then we'd have something like$$\frac{\partial \mathcal{L}}{\partial \phi} = -m\phi$$and$$\begin{align*}

\partial_{\sigma} \frac{\partial \mathcal{L}}{\partial(\partial_{\sigma} \phi)} =

\partial_{\sigma} \left[ \frac{1}{2} g^{\mu \nu} \cdot 2 \frac{\partial(\partial_{\mu} \phi)}{\partial(\partial_{\sigma} \phi)} \partial_{\nu} \phi \right] &= \partial_{\sigma} \left[ g^{\mu \nu} \delta^{\sigma}_{\mu} \partial_{\nu} \phi \right] \\

&= \partial_{\mu}(g^{\mu \nu} \partial_{\nu} \phi) \\

&= \partial^{\nu} \partial_{\nu} \phi + \left(\partial_{\mu} g^{\mu \nu}\right) \left(\partial_{\nu} \phi \right)

\end{align*}$$So EL equations give$$-m\phi = \partial^{\nu} \partial_{\nu} \phi + \left(\partial_{\mu} g^{\mu \nu}\right) \left(\partial_{\nu} \phi \right)$$Is that completely wrong? 🤭
 
  • #19
etotheipi said:
I tried, given the covariant derivative of a scalar field is just a partial derivative, and using the result we already worked for @TSny's equation [1], $$\begin{align*}

\nabla_{\mu} \nabla^{\mu} \phi = \nabla_{\mu} \partial^{\mu} \phi &= \frac{1}{\sqrt{|g|}} \partial_{\mu} \left(

\sqrt{|g|} \partial^{\mu} \phi \right) \\

&= \frac{1}{\sqrt{|g|}} \partial_{\mu} \left(\sqrt{|g|} g^{\mu \nu} \partial_{\nu} \phi \right)\end{align*}$$
Looks good to me.

Also, one notational question, do we define ##g \equiv \text{det}[(g_{\mu \nu})]##, and then I guess ##|g|## would just be the modulus, which we need to use when we have a Lorentzian metric?
Yes

I haven't learned anything about field theory so I'm probably going to get a lot of this wrong, but here's what I tried. I don't know what it means to write it in general covariant form
The non-covariant form is
$$A=\int \mathrm{d}^4 x\left[ \frac{1}{2} (\partial_{\mu} \phi)(\partial^{\mu} \phi)-\frac{m}{2} \phi^2\right]$$
As you noted, the integrand is already covariant due to ##\nabla_{\nu} \phi = \partial_{\nu} \phi##. However, the volume element ##d^4x## is not covariant. But this can be fixed by replacing ##d^4x## by ##d^4x \sqrt{|g|}##, which is covariant. That is, in switching from coordinates ##x^{\mu}## to ##\overline x^{\mu}##, you find ##d^4x \sqrt{|g|} = d^4 \overline x \sqrt{|\overline g|}##. The covariant form of ##A## then turns out to be $$A=\int \mathrm{d}^4 x \sqrt{|g|} \left[\frac{1}{2} (\partial_{\mu} \phi)(\partial^{\mu} \phi)-\frac{m}{2} \phi^2 \right] = \int \mathrm{d}^4 x \left[\sqrt{|g|} \frac{1}{2} (\partial_{\mu} \phi)(\partial^{\mu} \phi)-\sqrt{|g|}\frac{m}{2} \phi^2 \right] $$ You can then apply the Euler-Lagrange equations to the quantity in brackets on the far right.
 
Last edited:
  • Like
Likes vanhees71, JD_PM and etotheipi
  • #20
Thank you, that's pretty interesting! So covariance in this context means that the volume element is an invariant. I'll try to check the explicit transformation later. I'm not sure about what I did$$\frac{\partial \mathcal{L}}{\partial \phi} = \nabla_{\mu} \left(\frac{\partial \mathcal{L}}{\partial(\nabla_{\mu} \phi)} \right) \equiv \nabla_{\mu} \left(\frac{\partial \mathcal{L}}{\partial(\partial_{\mu} \phi)} \right)$$The LHS is fine,$$\frac{\partial \mathcal{L}}{\partial \phi} = - \sqrt{|g|} m\phi$$The RHS is$$\begin{align*}

\nabla_{\mu} \left( \frac{\partial \mathcal{L}}{\partial(\partial_{\mu} \phi)} \right) &=
\nabla_{\mu} \left( \sqrt{|g|} g^{\mu \nu} \partial_{\nu} \phi \right) \\

&= \partial_{\mu} \left( \sqrt{|g|} g^{\mu \nu} \partial_{\nu} \phi \right) + \frac{\sqrt{|g|} g^{\mu \nu} \partial_{\nu} \phi}{\sqrt{|g|}} \partial_{\mu} \left(\sqrt{|g|} \right) \\

&= 2\partial_{\mu} \left( \sqrt{|g|} g^{\mu \nu} \partial_{\nu} \phi \right) - \sqrt{|g|} \partial^{\nu} \partial_{\nu} \phi \quad \overset{!}{=} - \sqrt{|g|} m\phi

\end{align*}$$So$$\partial^{\nu} \partial_{\nu} \phi \equiv \partial_{\nu} \partial^{\nu} \phi = m \phi + \frac{2}{\sqrt{|g|}}\partial_{\mu} \left( \sqrt{|g|} g^{\mu \nu} \partial_{\nu} \phi \right) $$Going to be honest, I don't know where I'm going with this, since from the sounds of it the derivation should be straightforward...

I'll try again in the morning anyway, because I'm going to go to sleep early today ?:)
 
  • #21
The point is that you have
$$\partial_{\mu} \frac{\partial \mathcal{L}}{\partial_{\partial_{\mu} \phi}}=\frac{\partial \mathcal{L}}{\partial \phi}$$
with the generally covariant action principle, where
$$\mathcal{L}=\sqrt{|g|} \left [\frac{1}{2} g^{\mu \nu} (\partial_{\mu} \phi) (\partial_{\nu}\phi)-\frac{m^2}{2} \phi^2 \right].$$
The variational principle doesn't care about covariant derivatives, i.e., the EL equations are with the partial derivatives. If the action is generally covariant you get of course generally covariant equations.
 
  • Like
Likes JD_PM, TSny and etotheipi
  • #22
Thanks for the tips! Unfortunately, I'm still slightly too stupid to pick up on the hints you're dropping 😩. I guessed the other thing I could try would be to think about if the metric depends explicitly on ##\phi## and ##\partial_{\mu} \phi##, i.e. whether these terms have non-zero partial derivatives.$$\mathcal{L} = \frac{1}{2} \sqrt{|g|} g^{\mu \nu} (\partial_{\mu} \phi)(\partial_{\nu} \phi) - \frac{1}{2} m \sqrt{|g|} \phi^2$$With respect to ##\phi##,$$\frac{\partial \mathcal{L}}{\partial \phi} = -m\sqrt{|g|} \phi - \frac{1}{2} m \phi^2 \partial_{\phi} \left( \sqrt{|g|} \right) + \frac{1}{2} (\partial_{\mu} \phi) (\partial_{\nu} \phi) \partial_{\phi} \left( \sqrt{|g|} g^{\mu \nu} \right)$$With respect to ##\partial_{\sigma} \phi##,$$
\frac{\partial \mathcal{L}}{\partial(\partial_{\sigma} \phi)} = \sqrt{|g|} g^{\sigma \nu} (\partial_{\nu} \phi) + \frac{1}{2} (\partial_{\mu} \phi)(\partial_{\nu} \phi) \partial_{(\partial_{\sigma} \phi)} \left( \sqrt{|g|} g^{\mu \nu} \right) - \frac{1}{2}m \phi^2 \partial_{(\partial_{\sigma} \phi)} \left( \sqrt{|g|} \right)
$$With respect to ##q^{\sigma}##,$$\begin{align*}

\partial_{\sigma} \left( \frac{\partial \mathcal{L}}{\partial(\partial_{\sigma} \phi)} \right)

= \partial_{\sigma} \left( \sqrt{|g|} g^{\sigma \nu} (\partial_{\nu} \phi) \right)

&+ \frac{1}{2}(\partial_{\mu} \phi)(\partial_{\nu} \phi) \partial_{\sigma} \partial_{(\partial_{\sigma} \phi)}\left( \sqrt{|g|} g^{\mu \nu} \right) \\

&- \frac{1}{2}m \phi^2 \partial_{\sigma} \partial_{(\partial_{\sigma} \phi)} \left( \sqrt{|g|} \right) \\

&- m\phi (\partial_{\sigma} \phi) \partial_{(\partial_{\sigma} \phi)}\left( \sqrt{|g|} \right)\end{align*}
$$When I look to try and insert these into the EL condition for the extremum, I got stuck again because I don't know how to deal with ##\partial_{\sigma} \partial_{(\partial_{\sigma} \phi)}## and ##(\partial_{\sigma} \phi)\partial_{(\partial_{\sigma} \phi)}##.

Anyway sorry I couldn't get any further, if this is still too far off then it's probably better that I try and do some even more basic questions first, because I don't think there's much else you can say without giving away the answer :smile:
 
  • #23
The metric doesn't depend on ##\phi##. So, ## \partial_{\phi} \left( \sqrt{|g|}\right) = 0##, etc.

There was a typo in the original expression for ##\mathcal{L}##. The mass ##m## should be squared.
 
  • Like
  • Informative
Likes vanhees71, JD_PM and etotheipi
  • #24
TSny said:
The metric doesn't depend on ##\phi##. So, ## \partial_{\phi} \left( \sqrt{|g|}\right) = 0##, etc.

There was a typo in the original expression for ##\mathcal{L}##. The mass ##m## should be squared.
But doesn't that just give$$-m^2 \phi = \frac{1}{\sqrt{|g|}} \partial_{\mu} \left( \sqrt{|g|} g^{\mu \nu} \partial_{\nu} \phi \right)$$which is what we want on the RHS but definitely not what we want on the LHS?
 
  • #25
etotheipi said:
But doesn't that just give$$-m^2 \phi = \frac{1}{\sqrt{|g|}} \partial_{\mu} \left( \sqrt{|g|} g^{\mu \nu} \partial_{\nu} \phi \right)$$which is what we want on the RHS but definitely not what we want on the LHS?
The left side looks good. Are you worried about the negative sign on the left? That could be due to the choice of sign convention for the metric used by @vanhees71 when he wrote the Lagrangian. (+, -. -, -) vs. (-, +, +, +).
 
  • Like
Likes vanhees71 and etotheipi
  • #26
I'm just a bit confuzzled, because I thought the left hand side is supposed to be ##\nabla_{\mu} \nabla^{\mu} \phi##. I'm not sure how I can extract the required identity from this last equation, I guess that's why I tried that strange (incorrect) form of the EL equations with the covariant derivative a few posts back, in order to get it to show up.
 
  • #27
Oh, okay, I went to look at David Tong's QFT notes and on the first few pages he introduced the Klein-Gordon equation$$\square \phi = -m^2 \phi$$which is clearly what we want. Thanks for the help everyone! It's always fun to learn a little bit more than one expected to :smile:. Now if you'll excuse me, I'm just going to go cry for a little bit 😂
 
  • Like
Likes vanhees71
  • #28
etotheipi said:
I'm just a bit confuzzled, because I thought the left hand side is supposed to be ##\nabla_{\mu} \nabla^{\mu} \phi##. I'm not sure how I can extract the required identity from this last equation, I guess that's why I tried that strange (incorrect) form of the EL equations with the covariant derivative a few posts back, in order to get it to show up.
Ok, I think I see what's bothering you. You were hoping that the EL equation for the Lagrangian would be a way to prove the identity $$\nabla_{\mu} \nabla^{\mu} \phi = \frac{1}{\sqrt{|g|}} \partial_{\mu} \left( \sqrt{|g|} g^{\mu \nu} \partial_{\nu} \phi \right)$$.

But, instead you find that EL yields $$-m^2 \phi = \frac{1}{\sqrt{|g|}} \partial_{\mu} \left( \sqrt{|g|} g^{\mu \nu} \partial_{\nu} \phi \right)$$

You already proved the first identity at the beginning of post #18. But you were expecting the Lagrangian approach to be another way to derive it.

All right, fair enough. You don't get the desired identity directly from EL. But if we assume that the generally covariant form of @vanhees71 's Lagrangian is correct for the Klein-Gordon field in curved spacetime, then the equation of motion must be the generally covariant form of the Klein-Gordon equation. In Minkowski spacetime, the Klein-Gordon equation is ##\partial_{\mu} \partial^{\mu} \phi = -m^2 \phi##. The generally covariant form of this in curved spacetime is ##\nabla_{\mu} \nabla^{\mu} \phi = -m^2 \phi##. Comparing this to what we got from EL, we can make the identification that ##\nabla_{\mu} \nabla^{\mu} \phi = \frac{1}{\sqrt{|g|}} \partial_{\mu} \left( \sqrt{|g|} g^{\mu \nu} \partial_{\nu} \phi \right)##.

I'm not sure if this helps.
 
  • Like
  • Love
Likes vanhees71, JD_PM and etotheipi
  • #30
Oh hang on, I just realized that the ##\nabla^2## here only acts on spatial components, and$$\phi_{tt} - c^2 \nabla^2 \phi + m^2 c^4 \phi = 0$$is equivalent to$$\square \phi = -m^2 \phi$$so we are certainly not using a special case where ##\phi_{tt} = 0##. Sorry!
 
  • Like
Likes TSny and JD_PM
  • #31
etotheipi said:
Anyway, guess I better study more.
Don't push yourself too much. You're doing amazingly well! :oldsmile:
 
  • Like
  • Love
Likes PhDeezNutz, vanhees71, etotheipi and 1 other person
  • #32
TSny said:
Ok, I think I see what's bothering you. You were hoping that the EL equation for the Lagrangian would be a way to prove the identity $$\nabla_{\mu} \nabla^{\mu} \phi = \frac{1}{\sqrt{|g|}} \partial_{\mu} \left( \sqrt{|g|} g^{\mu \nu} \partial_{\nu} \phi \right)$$.
If you want to do this an alternative way is to couple the field to an external scalar current ##J##:
$$\mathcal{L}=\frac{1}{2} \sqrt{|g|} g^{\mu \nu} (\partial_{\mu} \phi)(\partial_{\nu} \phi)-\sqrt{|g|} J \phi.$$
Then you get
$$\Box \phi=J$$
as an equation of motion, and the variational principle tells you
$$\partial_{\mu} \frac{\partial \mathcal{L}}{\partial (\partial_{\mu} \phi)}=\frac{\partial \mathcal{L}}{\partial \phi}.$$
Now
$$ \frac{\partial \mathcal{L}}{\partial (\partial_{\mu} \phi)}=\sqrt{|g|} g^{\mu \nu} \partial_{\nu} \phi$$
and
$$\frac{\partial \mathcal{L}}{\partial \phi}=\sqrt{|g|} J,$$
from which
$$\partial_{\mu} (\sqrt{|g|} g^{\mu \nu} \partial_{\nu} \phi)=\sqrt{|g|} J \; \Rightarrow \; J=\Box \phi =\frac{1}{\sqrt{|g|}} \partial_{\mu} (\sqrt{|g|} g^{\mu \nu} \partial_{\nu} \phi).$$
QED
 
  • Like
Likes etotheipi
  • #33
Hello! I was rereading this nice thread and wanted to discuss a bit further (if you are willing to do so as well of course! :smile:)

Mmm I am actually trying to show Maxwell's equations in vacuum i.e. ##g^{\nu \mu}\nabla_{\mu} F_{\nu \sigma} = 0## and ##\nabla_{[\mu}F_{\nu \sigma]}=0## using the same Riemann tensor property i.e. ##\nabla_{\mu} \nabla_{\nu} A_{\sigma} = R_{\sigma \nu \mu \rho} A^{\rho}##, where ##F_{\mu \nu} = \nabla_{\mu} A_{\nu} - \nabla_{\nu} A_{\mu}##.

Let's focus on ##g^{\nu \mu}\nabla_{\mu} F_{\nu \sigma} = 0## first.

We get

\begin{align*}
\nabla_{\mu} F_{\nu \sigma} &= \nabla_{\mu} \nabla_{\nu} A_{\sigma} - \nabla_{\mu} \nabla_{\sigma} A_{\nu} \\
&= R_{\sigma \nu \mu \rho} A^{\rho} - R_{\nu \sigma \mu \rho} A^{\rho} \\
&= 2R_{\sigma \nu \mu \rho} A^{\rho}
\end{align*}

Multiplying both sides by ##g^{\nu \mu}## yields

\begin{equation*}
\nabla^{\nu} F_{\nu \sigma} = 2 R_{\sigma \rho} A^{\rho} = 0
\end{equation*}

Where I used the fact that, in vacuum, ##R_{\mu \nu} = 0##.

Does this proof look correct to you? :smile:

I am stuck on how to show that ##\nabla_{[\mu}F_{\nu \sigma]}=0## by means of ##\nabla_{\mu} \nabla_{\nu} A_{\sigma} = R_{\sigma \nu \mu \rho} A^{\rho}## though

I start expanding it out

\begin{align*}
\nabla_{[\mu } F_{\nu \rho]} &= \frac{1}{3!} \left( \nabla_{\mu} F_{\nu \rho} + \nabla_{\nu} F_{\rho \mu} + \nabla_{\rho} F_{\mu \nu} -\nabla_{\mu} F_{\rho \nu} -\nabla_{\nu} F_{\mu \rho} -\nabla_{\rho} F_{\nu \mu} \right) \\
&= \frac{1}{3} \left( R_{\rho \nu \mu \sigma} + R_{\mu \rho \nu \sigma} + R_{\nu \mu \rho \sigma} - R_{\nu \rho \mu \sigma} - R_{\rho \mu \nu \sigma} - R_{\mu \nu \rho \sigma}\right) A^{\sigma} \\
&= \frac{2}{3} \left( R_{\rho \nu \mu \sigma}+ R_{\mu \rho \nu \sigma} + R_{\nu \mu \rho \sigma}\right) A^{\sigma}
\end{align*}

But I do not see how to show that the last equation vanishes without having to explicitly write the Riemann tensor as a function of Christoffel symbols (which does not seem to be the best idea 😅).Regards.

JD.
 
  • Like
Likes etotheipi
  • #34
JD_PM said:
\begin{align*}
\nabla_{[\mu } F_{\nu \rho]} &= \frac{1}{3!} \left( \nabla_{\mu} F_{\nu \rho} + \nabla_{\nu} F_{\rho \mu} + \nabla_{\rho} F_{\mu \nu} -\nabla_{\mu} F_{\rho \nu} -\nabla_{\nu} F_{\mu \rho} -\nabla_{\rho} F_{\nu \mu} \right) \\
&= \frac{1}{3} \left( R_{\rho \nu \mu \sigma} + R_{\mu \rho \nu \sigma} + R_{\nu \mu \rho \sigma} - R_{\nu \rho \mu \sigma} - R_{\rho \mu \nu \sigma} - R_{\mu \nu \rho \sigma}\right) A^{\sigma} \\
&= \frac{2}{3} \left( R_{\rho \nu \mu \sigma}+ R_{\mu \rho \nu \sigma} + R_{\nu \mu \rho \sigma}\right) A^{\sigma}
\end{align*}

But I do not see how to show that the last equation vanishes without having to explicitly write the Riemann tensor as a function of Christoffel symbols (which does not seem to be the best idea 😅).

Alright, I got it.

\begin{equation*}
R_{[\rho \nu \mu]\sigma} = 0
\end{equation*}

😅
 
  • Like
Likes etotheipi
  • #35
Just for fun:$$
\begin{align*}

\mathrm{d} \star F &= \star j \\

0 = \mathrm{d}\mathrm{d}\star F &= \mathrm{d} \star j \implies \int_{\Omega} \mathrm{d} \star j = \oint_{\partial \Omega} \star j = 0

\end{align*}$$
 
  • Like
  • Love
Likes vanhees71 and JD_PM
<h2>1. What is charge conservation in a curved spacetime?</h2><p>Charge conservation in a curved spacetime refers to the principle that the total electric charge within a given region of spacetime remains constant over time. This means that the amount of positive charge must always equal the amount of negative charge within a closed system.</p><h2>2. How does charge conservation apply to curved spacetime?</h2><p>In curved spacetime, the geometry of space and time is affected by the presence of massive objects. This can cause changes in the electric field and therefore impact the distribution of electric charge. However, the total amount of charge within a given region of spacetime must remain constant, even if the distribution of charge is altered.</p><h2>3. Why is charge conservation important in curved spacetime?</h2><p>Charge conservation is important in curved spacetime because it is a fundamental principle of electromagnetism. It helps to explain the behavior of electrically charged particles in the presence of massive objects and is essential for understanding the dynamics of the universe on a large scale.</p><h2>4. How is charge conserved in a curved spacetime?</h2><p>Charge is conserved in a curved spacetime through the laws of electromagnetism, specifically Gauss's law and Faraday's law. These laws state that the electric flux through a closed surface and the circulation of the electric field around a closed loop must be equal to the total charge enclosed within that surface or loop, respectively.</p><h2>5. Can charge conservation be violated in a curved spacetime?</h2><p>No, charge conservation is a fundamental principle of electromagnetism and is considered to be a law of nature. It has been extensively tested and has never been found to be violated in any observed phenomenon, including in curved spacetime. Any apparent violations can be explained by other factors, such as the creation or annihilation of particles.</p>

1. What is charge conservation in a curved spacetime?

Charge conservation in a curved spacetime refers to the principle that the total electric charge within a given region of spacetime remains constant over time. This means that the amount of positive charge must always equal the amount of negative charge within a closed system.

2. How does charge conservation apply to curved spacetime?

In curved spacetime, the geometry of space and time is affected by the presence of massive objects. This can cause changes in the electric field and therefore impact the distribution of electric charge. However, the total amount of charge within a given region of spacetime must remain constant, even if the distribution of charge is altered.

3. Why is charge conservation important in curved spacetime?

Charge conservation is important in curved spacetime because it is a fundamental principle of electromagnetism. It helps to explain the behavior of electrically charged particles in the presence of massive objects and is essential for understanding the dynamics of the universe on a large scale.

4. How is charge conserved in a curved spacetime?

Charge is conserved in a curved spacetime through the laws of electromagnetism, specifically Gauss's law and Faraday's law. These laws state that the electric flux through a closed surface and the circulation of the electric field around a closed loop must be equal to the total charge enclosed within that surface or loop, respectively.

5. Can charge conservation be violated in a curved spacetime?

No, charge conservation is a fundamental principle of electromagnetism and is considered to be a law of nature. It has been extensively tested and has never been found to be violated in any observed phenomenon, including in curved spacetime. Any apparent violations can be explained by other factors, such as the creation or annihilation of particles.

Similar threads

  • Advanced Physics Homework Help
Replies
4
Views
2K
  • Advanced Physics Homework Help
Replies
3
Views
4K
  • Advanced Physics Homework Help
Replies
2
Views
2K
  • Special and General Relativity
Replies
21
Views
1K
  • Advanced Physics Homework Help
2
Replies
42
Views
7K
  • Special and General Relativity
Replies
7
Views
1K
  • Special and General Relativity
Replies
1
Views
735
  • Advanced Physics Homework Help
Replies
1
Views
810
  • Advanced Physics Homework Help
Replies
3
Views
1K
  • Sticky
  • Special and General Relativity
Replies
15
Views
6K
Back
Top